Trig Differentiation for tan2(4x): Solving 1+tan2(4x) using the Chain Rule

Click For Summary
The differential of the function 1 + tan²(4x) is derived using the chain rule, resulting in 8tan(4x)sec²(4x). The constant 1 differentiates to 0, allowing focus on tan²(4x), which differentiates to 2tan(4x)sec²(4x) multiplied by the derivative of the inner function, 4x. The correct application of the chain rule reveals that the coefficient should be 32, not 8, due to the additional factor from differentiating 4x. The final expression confirms the correct differentiation process and application of the chain rule. Understanding these steps is crucial for accurately solving trigonometric differentiation problems.
jsmith613
Messages
609
Reaction score
0

Homework Statement



how is the differential of 1+tan2(4x)

8*tan(4x)*sec2(4x)

Homework Equations


The Attempt at a Solution


So 1 differentiates to 0 (we can now ignore this)
tan2(4x) differentiates to 2*tan(4x)*sec2(4x)?

BRING POWER FORWARD, DOWN POWER BY 1, DIFFERENTIATE TERM IN BRACKET
by the chain rule?
 
Physics news on Phys.org
jsmith613 said:
BRING POWER FORWARD, DOWN POWER BY 1, DIFFERENTIATE TERM IN BRACKET
by the chain rule?


You stated it in your question, but never differentiated the term inside inside the bracket.

2 \cdot tan(4x)sec^{2}(4x) \cdot \frac{d}{dx} (4x)~=~
 
Last edited:
hold on
yours goes to 32...
NOT 8...
 
jsmith613 said:
hold on
yours goes to 32...
NOT 8...

My mistake :-p I fixed it in my first post ( coefficient was originally wrong), but my point still holds. You ALSO need to differentiate the 4x.
 
oh of course
differentiate u2 = 2u
differentiate sec2(4x) is 4*sec(4x)*tan(4x)
multiply differntials to give
2*sec(4x)*4*sec(4x)*tan(4x)
 
Exactly!:approve:
 
Question: A clock's minute hand has length 4 and its hour hand has length 3. What is the distance between the tips at the moment when it is increasing most rapidly?(Putnam Exam Question) Answer: Making assumption that both the hands moves at constant angular velocities, the answer is ## \sqrt{7} .## But don't you think this assumption is somewhat doubtful and wrong?

Similar threads

  • · Replies 5 ·
Replies
5
Views
1K
  • · Replies 5 ·
Replies
5
Views
2K
  • · Replies 13 ·
Replies
13
Views
13K
  • · Replies 2 ·
Replies
2
Views
2K
  • · Replies 1 ·
Replies
1
Views
1K
  • · Replies 4 ·
Replies
4
Views
1K
  • · Replies 1 ·
Replies
1
Views
2K
  • · Replies 2 ·
Replies
2
Views
1K
Replies
10
Views
2K
  • · Replies 4 ·
Replies
4
Views
2K